If angular momentum is conserved in a system whose moment of inertia is decreased, Will its rotational kinetic energy be also conserved? Explain.

L= Iw
where ,
L is angular momentum.
I is moment of Inertia
w is angular momentum.
I1w1 = I2w2
or I1w1/I2w2 = 1   ............(1)
Now lets check for Rotaional Kinetic energy 
1/2 I1​w1​2 / 1/2 I2 w22  

=I1w12I2w22 = w1w2  1 
so ​rotational kinetic energy not conserved


 

  • -11
What are you looking for?